Lean Six Sigma Green Belt Practice Questions

Réussis tes devoirs et examens dès maintenant avec Quizwiz!

When making inferences about a population variance based on a single sample from that population, what distribution is used? a. Chi-square b. Normal c. t distribution d. F distribution

a. Chi-square

Consider the following network with events marked within the circles and durations in weeks. The critical path is: a. 1-3-6-8-10 b. 1-3-6-9-10 c. 1-4-6-8-10 d. 1-4-6-9-10

c. 1-4-6-8-10

The target length of an initial Six Sigma project should be approximately: a. 60 days b. 90 days c. 120 days d. 180 days

c. 120 days

Using Six Sigma methodology, a company at 4.5 sigma would have a failure rate of: a. 3.4 ppm b. 233 ppm c. 1350 ppm d. 6210

c. 1350 ppm The Six Sigma approach allows for a +/- 1.5 sigma shift. Answer b represents the 5 sigma level and answer d the 4 sigma level.

It is desirable to reduce the variation in a process. The current variation is known to be six. A new process yielded a standard deviation of two for twenty-five trials. What is the chi-square calculated statistic? a. 13.85 b. 15.66 c. 16.00 d. 18.24

c. 16.00

In the figure below, what is the intersection of events A and C? a. 1, 3, and 4 b. 1, 2, 3, 4, 5, 7, and 8 c. 3 and 4 d. 2, 3, 4, and 7

c. 3 and 4

If a one-tail F test (95% confidence) with ten samples yielded a variance of nine, and nine samples yielded a variance of four, what F critical value would be used? a. 3.23 b. 3.44 c. 3.39 d. 3.14

c. 3.39

The DPMO for a process is 860. What is the approximate Six Sigma level of the process? a. 4.2 b. 4.4 c. 4.6 d. 4.8

c. 4.6

Given the data below, what is the 90% confidence interval for the variance? 22, 23, 19, 17, 29, 25 a. 4.21 - 99.07 b. 15.32 - 28.66 c. 8.27 - 79.88 d. 16.87 - 56.52

c. 8.27 - 79.88

A scatter diagram is used to plot gas mileage versus the weight of a car. One would expect the shape of the resulting plot to be: a. A low positive b. An increasing slope c. A decreasing slope d. No correlation display

c. A decreasing slope

The critical path in a project means: a. The project is important to the profits of the organization b. Slack times can be used to delay the ending date of the project c. Delays of events on this path delay the ending date of the project d. Activities cannot be crashed

c. Delays of events on this path delay the ending date of the project

Which of the following process mapping symbols would be associated with a decision point? a. Oval b. Rectangle c. Diamond d. Square

c. Diamond

Since many variables are important in control charting, what is the risk of having an operator plot a large number of characteristics? a. None, if the operator is trained and knowledgeable b. Danger if overlooking a CTQ characteristic c. Distraction from the actual process itself d. It is non-value added work in the lean philosophy

c. Distraction from the actual process itself

Which of the following DOE strategies most resembles the Kaizen philosophy? a. Response surface b. Mixture designs c. EVOP d. One factor at a time analysis

c. EVOP

One way communications within an organization would include: a. Upward and downward b. Horizontal and informal c. Emails and letters d. Rumors and gossip

c. Emails and letters

Good improvement team members will: a. Provide valid excuses when they miss a meeting b. Agree with the team even when it is wrong c. Encourage participation by other team members d. Withhold unpopular information from the team

c. Encourage participation by other team members

The test used for testing significance in an analysis of variance table is the: a. Z test b. t test c. F test d. Chi-square test

c. F test

When performing calculations on sample data: a. A continuous relative frequency graph is created b. Rounding the data has no effect on the mean and standard deviation c. Coding the data has no effect on the mean and standard deviation d. Coding and rounding affect both the mean and standard deviation

d. Coding and rounding affect both the mean and standard deviation

All multi-vari charts would initially plot a measurement for which of the following categories? a. Within batch b. Top to bottom c. Positional d. Cylindrical

c. Positional

Identify the business element that would directly reduce cycle time: a. Utilizing problem solving tools b. Implementing 5S c. Practicing SMED d. Supervising people appropriately

c. Practicing SMED Supervising people might not reduce cycle time. 5S and utilizing problem solving tools could indirectly reduce cycle time. SMED (single minute exchange of dies) will directly reduce cycle time.

One would consider mistake proofing to be in what cost category? a. Appraisal b. Internal failure c. Prevention d. Overhead

c. Prevention

Of the four classifications of quality costs, which will prove the greatest dollar savings versus dollars spent? a. Appraisal costs b. Internal failure costs c. Prevention costs d. External failure costs

c. Prevention costs

Maintaining extra copies of instructional information in description of which NVA? a. Overproduction b. Inventory c. Processing d. Transport

c. Processing The most likely culprit is processing muda. There might be some overproduction and waiting issues as well.

Takt time takes on a great importance in: a. Cause-and-effect diagrams b. 5S housekeeping organization c. Poke-Yoke/mistake proofing applications d. Continuous flow manufacturing

d. Continuous flow manufacturing

A student's t test can be used to determine whether or not differences exist in: a. Variability b. Confidence intervals c. Correlation coefficients d. Averages

d. Averages

When selecting and scaling the process input variables for an experiment, which of the following is a desirable approach? a. Include as many factors as possible b. Set factor levels at extreme levels c. Combine process measurement responses when possible d. Be bold, but not foolish, in selecting high and low factor levels

d. Be bold, but not foolish, in selecting high and low factor levels

Which of the following is considered muda? a. Design reviews b. Level scheduling of work c. Checking a die before installation d. Breakdowns in machinery

d. Breakdowns in machinery Design reviews are part of the continuous improvement effort and are a value added activity.

What inference test does not require some knowledge of a test or population variation? a. t test b. Paired t test c. z test d. Chi-square test

d. Chi-square test

When describing a process being considered for cycle time reduction, the existing process flows are known as: a. Key operation points (KOPS) b. Dynamic pull chart c. Current state map d. Activity network diagram (AND)

c. Current state map

A control chart is used to: a. Determine if defective parts are being produced b. Measure process capability c. Determine causes of potential variation d. Detect non-random variation in processes

d. Detect non-random variation in processes

Project improvement team members normally have: a. Narrow skills and experiences b. Diverse skills and narrow experiences c. Diverse experiences and narrow skills d. Diverse skills and experiences

d. Diverse skills and experiences

One would say that the final stage in project management would be: a. The project proposal b. The project review c. Lessons learned d. Document archiving

d. Document archiving

One thousand units of product were examined for the possibility of 5 different undesirable characteristics. A total of 80 defects were found. How many defects would be expected in a million opportunities? a. 16,000 b. 26,666 c. 61,458 d. 80,000

16,000

An organized and disciplined approach to problem solving in most six sigma organizations is called: a. SIPOC b. DMAIC c.. PDCA d. DPMO

B. DMAIC

The equation below is : a. The covariance of X and Y b. The correlation coefficient of X and Y c. The coefficient of determination of X and Y d. The variance of the product of X and Y

The correlation coefficient of X and Y

Six Sigma means 3.4 ppm considering a shift in the mean of 1.5 standard deviations. What is the value of Six Sigma without the 1.5 standard deviations shift in the mean? a. 0.002 ppm b. 0.135 ppm c. 2.4 ppm d. 4.9 ppm

a. 0.002

Fifteen process improvement ideas have been submitted for a project selection. Unknown to the team, to of these ideas have the potential for breakthrough improvement. If your team selects two projects at random, what are the chances of picking both winners? a. 0.0095 b. 0.0714 c. 0.1333 d. 0.1538

a. 0.0095

The car you drive has a 90% chance of starting in the morning. It is blocked by your spouse's car which has an 80% chance of starting. Both cars are blocked by your son's care which has a 70% chance of starting. What is the probability of getting to work in your own car? a. 0.504 b. 0.560 c. 0.630 d. 0.720

a. 0.504

A process is in control with p bar = 0.10 and n = 100. The three sigma limits of the np control chart are: a. 1 and 1.9 b. 9.1 and 10.9 c. 0.01 and 0.19 d. 0.07 and 0.13

a. 1 and 1.9

Determine the reported Ppk for a process with a spread of 10 units, an average of 23, and upper and lower specification limits of 15 and 35 respectively. a. 1.6 b. 1.8 c. 2.0 d. 0.625

a. 1.6

If a sample size of 16 yields an average of 12 with a standard deviation of 3, estimate the 95% confidence interval for the population (assume a normal distribution). a. 10.40 </= mu </= 13.60 b. 10.45 </= mu </= 13.55 c. 10.53 </= mu </= 13.46 d. 10.77 </= mu </= 13.23

a. 10.40 </= mu </= 13.60

Compute the upper control limit for an S chart based on a sample size of ten. If the process is in control with a mean of 40 an a standard deviation of 7. a. 12.0 b. 13.3 c. 15.7 d. 21.0

a. 12.0

It is desirable to reduce the variation in a process. The current variance is known to be seven. A new process yielded a standard deviation of two for twenty-five test trials. What is the calculated statistic and decision for 95% confidence? a. 13.71, reject the null b. 13.71 fail to reject the null c. 13.85, reject the null d. 13.85, fail to reject the null

a. 13.71, reject the null

What would be the calculated F statistic if a one-tail T test (95% confidence) with ten samples, yielding a variance of nine, and nine samples yielding a variance of four. a. 2.25 b. 3.39 c. 3.44 d. 5.06

a. 2.25

A random sample size n is to be taken from a large population having a standard deviation of 1 inch. The sample size is to be determined so that there will be a 0.05 risk probability of exceeding a 0.1 inch tolerance error in using the population mean to estimate sample mean. Which of the following values is the required sample size? a. 385 b. 44 c. 202 d. 109

a. 385

What is the mode of the following data set? 4, 2, 6, 8, 7, 4, 3, 4, 7 a. 4 b. 5 c. 6 d. 7

a. 4

Refer to the Venn diagram below: If the probability of event A is 20%, the probability of event B is 30%, and the probability of A intersecting event B is 8%, what is the probability of neither event? a. 58% b. 56% c. 48% d. 44%

a. 58%

Determine the coefficient of variation for the last 500 pilot plant test runs of high temperature film having a mean of 900 degrees Kelvin with a standard deviation of 54 degrees. a. 6% b. 16.7% c. 0.6% d. 31%

a. 6%

A designed experiment has been conducted at three levels (A, B, C) yielding the following "coded" data...As a major step in the analysis, one would calculate the degrees of freedom for the "error" sum of squares to be: a. 7 b. 9 c. 6 d. 3

a. 7

Which of the following is true in regards to blocking? a. A block is a dummy factory which doesn't interact with real factors b. A blocking factor has two levels c. The experiment is a subdivision of the block d. Blocks permit full randomization of runs

a. A block is a dummy factor which doesn't interact with real factors

Resistance to change resulting from an improvement project can cause failures. The project team might consider development of a plan to handle stakeholders. This plan is most likely to be termed: a. A communications plan b. A resistance plan c. A PERT chart d. A Gantt chart

a. A communications plan

A fractional factorial experiment resulted in the main effects and two factor interactions effects being confounded. The confounding can be resolved by running: a. A full factorial experiment with the same factors b. Changing the sign of the main effect c. Two or more replications of each trial d. Discarding the duplicated factors

a. A full factorial experiment with the same factors

If the means of two populations are close in value, then: a. The null hypothesis will not be rejected b. A large sample is necessary to reject the null hypothesis c. A null hypothesis decision will often be delayed d. The appropriate critical value is very important

a. A large sample is necessary to reject the null hypothesis

A chart that has defect types on the x-axis, customers on the y-axis, and numbers of defects at the corresponding column and row intersections is called: a. A matrix diagram b. A correlation chart c. A Pareto diagram d. A cause-and-effect diagram

a. A matrix diagram

Benchmarking might be defined as which of the following? a. A process for rigorously measuring one company's performance versus the best in class b. An acceptable standard of achievement c. Comparing a company's performance to a standard and then to another company's performance d. The search for the industry practices that lead to negative performance

a. A process for rigorously measuring one company's performance versus the best in class

What graphical data method can show the value of all individual readings? a. A stem and leaf plot b. A grouped probability density function c. A normal histogram d. A complex box plot

a. A stem and leaf plot

Which MSA methods allows the interaction between operators and parts to be determined? a. ANOVA method b. Average and Range method c. Interaction method d. Range method

a. ANOVA method

The new problem solving tool which incorporates PERT and CPM techniques into a project flow chart is called: a. Activity network diagram b. Prioritization matrix c. Tree diagram d. Process decision program chart

a. Activity network diagram

Lean enterprise may be summarized as: a. An entire organization involved with improvement b. Implementation of SMED cycle time techniques c. Poka-yoke techniques in action d. Ergonomic principles in the workplace

a. An entire organization involved with improvement It is important that the entire organization be committed to lean thinking. The other items are of lesser importance.

Variance from budget for a project: a. Are the differences between planned and actual costs b. Indicate the project manager did a poor job of controlling costs c. Are usually expressed in standard deviation units from the norm d. Are expected in most complex design activities

a. Are the differences between planned and actual costs

Why is the normality assumption essential to the interpretation of the capability index? a. Because a spread of 6 standard deviations represents 99.73% of cases b. Because only normal distributions are capable of statistical control c. Because the specifications are always explained by the bell curve d. Because the normal distribution always has a mean of 0

a. Because a spread of 6 standard deviations represents 99.73% of cases

The histogram below displays what type of distribution? a. Bimodal b. Polymodal c. Negative skewed d. Truncated

a. Bimodal

Assuming no interactions, the main effects of analysis of one-half factional factorial experiment compared with a comparable full factorial experiment, yields which of the following outcomes? a. Both results are approximately the same b. Only full factorial is accurate c. Both can be calculated but computer software is required. d. One-half factional factorial gives one-half of the final outcome

a. Both results are approximately the same

From an upper management perspective, what has been the principal motivating factor in embracing Six Sigma? a. Bottom line results b. Market share growth c. Defect reductions d. Customer focus

a. Bottom line results Phil Crosby once stated that upper management is interested in money, making money, and not losing money. He said there must be something else, but he never got that far in conversations with them. Answers b, c, and d, should be considered subsets of answer a.

Why has Six Sima been successful for many organizations? a. Bottom line results are enhanced b. A +/- 1.5 sigma shift is included c. An undisciplined approach is used d. Sound statistical approaches are eliminated

a. Bottom line results are enhanced The +/- 1.5 sigma shift is only a small part of Six Sigma. Answer b is thus eliminated. The opposite of answers c and d are true.

Which of the following items is the most likely candidate to assist the problem definition stage of Six Sigma? a. CTQ trees b. Metrics selection c. PERT charts d. Control charts

a. CTQ trees

A value stream map would normally provide data on: a. Changeover and cycle times b. Color coding techniques c. Measurement system analysis d. Supplier's finished goods inventory

a. Changeover and cycle times

When the natural process limits are greater than the specification range, which of the following courses of action involves the customer? a. Changing the specifications b. Centering the process c. Reducing the variability d. Accepting the losses

a. Changing the specifications

Some authorities contend that the most neglected phase in the DMAIC improvement process is: a. Control b. Define c. Measure d. Assess

a. Control

Which of the following is a measure of marketplace response? a. Customer courtesy ratings b. The cost of quality c. The percentage of defects generated d. A reduction in product or service variation

a. Customer courtesy ratings Customer courtesy ratings are a marketplace response. The other answers may be measurements of resource usage.

Six Sigma design uses which sequence in the creation of new products? a. Define, measure, analyze, design, and verify b. Design, define, analyze, verify, and measure c. Analyze, measure, define, verify, and design d. Measure, define, design, analyze, and verify

a. Define, measure, analyze, design, and verify Using all of the steps provided, the sequence would be define, measure, analyze, design, and verify. This is per Simon (2000).

The most effective and efficient method of solving quality problems for a product is to concentrate efforts in the area of: a. Design b. Production c. Quality improvement d. Lean techniques

a. Design It has been stated that 80% of the quality problems are design related. Given this fact, the best option is to prevent design problems from occuring.

A process map is ideal for: a. Detecting the causes for delays b. Determining defective parts c. Prioritizing problems d. Determining if two variables are related

a. Detecting the cause for delays

The implementation of a control plan in the painting department was so successful that a master black belt wants you to implement the same control plan in the final assembly line. What changes should be made before implementing the necessary control plan? a. Each control plan is unique, a new plan must be developed for the final assemble line b. A lot of changes will be necessary c. No changes are needed, since the two areas are in the same company d. Minor changes may be necessary to reflect the assembly line differences

a. Each control plan is unique, a new plan must be developed for the final assemble line

Upon completion of a FMEA, what critical event should be established next? a. Ensure that a process owner is in charge of the FMEA b. Designate an engineer to follow-up on the recommended actions c. Review the documents at regularly scheduled interviews d. Have cross functional teams assigned to each FMEA for review activities

a. Ensure that a process owner is in charge of the FMEA Once a FMEA has been completed, the overall ownership must be turned over to a process owner who includes it as a part of his/her responsibilities. Answer b is not an ideal choice since the engineer may or may not be a process owner and could be reassigned from time to time. Answer c is important, but it is usually not the next event after completion of a FMEA. Cross functional teams are needed to conduct FMEAs, but assigning a team to review activities may not be practical.

Which of the following statements is most applicable to trend analysis? a. Experience is required for proper interpretation b. Bar charts are more informative than run charts c. Most applications should be reflected as an improvement percentage d. An improving trend is an indication of corporate survival

a. Experience is required for proper interpretation

Ratios of two variances drawn from the same normal population are described by which one of the following distributions: a. F b. Student's t c. Chi-square d. Normal

a. F

For a one-tail F test (95% confidence) with ten samples yielding a variance of nine and nine samples yielding a variance of four, what would be the test result? a. Fail to reject the null hypothesis b. Reject the null hypothesis c. Accept the alternate hypothesis d. Fail to reject the alternate hypothesis

a. Fail to reject the null hypothesis

A value stream map that describes the process after lean principles have been applied is called a: a. Future state map b. Ideal state map c. Activity network diagram d. Optimized process chart (OPC)

a. Future state map

Precision is: a. Getting consistent results repeatedly b. Reading to one decimal greater than the reported dimension c. Distinguishing small deviations from a standard value d. Extreme care in the analysis of data

a. Getting consistent results repeatedly

Excessive conflict within an improvement team: a. Has a negative effect on team members and should be avoided b. Has a positive effect on creating alternate solutions c. Most often results in win-win situations d. Promotes equal participation among members

a. Has negative effect on team members and should be avoided

Manual project management planning and controlling methods: a. Have an advantage over computer methods in terms of cost b. Have become nearly extinct because of the availability of computer methods c. Are best for large, complex projects d. Are harder to learn than computer methods

a. Have an advantage over computer methods in terms of cost

The team champion has this position because: a. He or she is at a high level in the organization b. He or she finished the project first c. His or her team saved the most money d. The team members elected him or her

a. He or she is at a high level in the organization

If one is interested in reducing inventory and product lead times, what Japanese control term would apply? a. Heijunka b. Seiketsu c. Seiso d. Seiri

a. Heijunka

Which of the following are DFSS techniques? a. IDOV and DMADV b. Kaizen and kanban c. SPC and CWQC d. DPIVs and KPOVs

a. IDOV and DMADV IDOV and DMADV are DFSS techniques. The other answer choices are different tools.

Which IDOV phase defines overall product requirements, quantifies marketing information and customer feedback, and creates the first set of CTQ (critical to quality) features? a. identity b. design c. optimize d. validate

a. Identify The definition clearly refers to the identify phase of the IDOV methodology.

When designing for TPM, why would BITE (built-in test equipment) be of advantage? a. In fault isolation b. For modularization c. In standardization d. For malfunction annunciation

a. In fault isolation

The alpha critical region, to determine if a new pen writes more strokes before refill would be placed: a. In the upper tail b. In both upper and lower tails c. In the lower tail d. In neither tail

a. In the upper tail

Which of the following is a goal of TOC? a. Increase throughput b. Increased inventory c. Increased operating expenses d. Flow greater than demand

a. Increase throughput Goldratt supported increasing throughput, reducing inventory and reducing operating expenses. He endorsed balancing the flow through the plant with customer demand.

A team sponsor or champion typically: a. Is a liaison between the team and upper management b. Attends all team meetings c. Directs the team on implementing solutions d. Fulfills the facilitator role

a. Is a liaison between the team and upper management

What is the importance of the reaction plan in a control plan? a. It describes what will happen if a key variable goes out of control b. It indicates that a new team must be formed to react to a problem c. It lists how often the process should be monitored d. It defines the special characteristics to be monitored

a. It describes what will happen if a key variable goes out of control

What is the best definition of takt time? a. It is a calculated time element that equals customer demand b. It is the speed at which parts must be manufactured in order to satisfy demand c. It is the heartbeat of any lean system d. It is the application of Kaizen to continuous flow manufacturing

a. It is the calculated time element that equals customer demand All of the answers to this question sound pretty neat, but the key elements in the answer must be some version of the net operating time period (a time element) divided by customer requirements per period (customer demand). Answer a is the best selection. The name takt is German for baton, implying a rhythm, pace or heartbeat for the system. However, answer c is meant to be a secondary disaster choice. The word time in the question eliminates both answers b and c.

Why is the PDCA cycle so readily accepted by most American teams and individuals? a. It is the natural way that most people already approach problems b. it was promoted by Dr. Deming who has a wide American following c. It has been widely used in Japan with success d. It requires much less work than comparable improvement techniques

a. It is the natural way that most people already approach problems Think about the PDCA process. When one approaches a problem, isn't it natural to plan for a potential solution, take some action, check the outcome, and take further action based on the result?

Which of the following is a correct description of QFD? a. It translates fuzzy customer requirements into design specifications b. It an be used to replaced FMEA c. It is similar to project management d. It does not permit comparisons with competitors

a. It translates fuzzy customer requirements into design specifications

The tool/technique widely used by a number of automotive suppliers to control material flow is called: a. Kanban b. Muda c. Poka-yoke d. An andon board

a. Kanban

In Japanese or American factories, supplying Japanese manufacturers, the flow of material is controlled by: a. Kanban cards b. 5S activities c. Production boards d. Jidhoka devices

a. Kanban cards

The Japanese techniques that complement the visual factory concept are: a. Kanban, poka-yoke, 5S b. Poka-yoke, muda, andon boards c. 5-whys, andon boards, TPM d. 5S, kanban, muda

a. Kanban, poka-yoke, 5S

A customer satisfaction program was started on the right foot and has gone very well for the last year or so. The company should: a. Look to improve the program with new customer input b. Do nothing with the program, it's not broken c. Form a manager's group to add new wrinkles to the program d. Concentrate on long-term customers

a. Look to improve the program with new customer input

Tool boards, jidhoka devices, and red lights all combine to: a. Make problems visible b. Prevent defective products c. Maintain management control d. Display targets for improvement

a. Make problems visible

The production of product in large lots has what advantage? a. Maximization of machine efficiencies b. Longer customer delivery lead times c. Additional product transportation expenses d. Potential product damage or deterioration costs

a. Maximization of machine efficiencies Answers b, c, and d are all disadvantages pointed out by proponents of lean manufacturing and continuous flow. Answer a is generally recognized as the main disadvantage of large batch runs that may be more than offset by the disadvantages.

Which of the following is a necessary assumption to validate the meaning of the standard deviation of the measurement variability? a. Measurement errors are independent b. The measurement scale is normally distributed c. Measurement errors are independent of the operations involved in the study d. Measurement errors are skewed in the direction of normality

a. Measurement errors are independent

Which of the following describes poorly functioning teams? a. Members act independently without inter-dependency b. Objectives are realistically set and met c. Team members listen to what is being said d. Facts and opinions are distinguished

a. Members act independently without inter-dependency

Any action taken to reduce the probability and/or consequences of an adverse outcome from a development project is called: a. Mitigation b. Transference c. Avoidance d. Acceptance

a. Mitigation

The measure of the central location for the nominal scale is considered to be the: a. Mode b. Median c. Arithmetic mean d. Average

a. Mode

After a screening experiment, all significant factors happen to be qualitative. A decision to run some intermediate values will result in: a. Nothing, it is not possible to optimize qualitative variables b. The discovery of the optimum combination of qualitative variables c. Indications of curvature around the zero setting d. The discovery of potential interactions between qualitative variables

a. Nothing, it is not possible to optimize qualitative variables

There are five fabricating operations that can be performed in any sequence. Four of the operations yield 98% recover and one yields 80% recovery. What is the preferred production sequence, assuming that 80% operation cannot be immediately improved? a. Place the 80% operation first b. Place the 80% operation in the middle c. Place the 80% operation last d. It does not matter; the RTY is the same

a. Place the 80% operation first

The distribution which has a mean equal to the variance is the: a. Poisson b. Exponential c. Weibull d. Rayleigh

a. Poisson

An attempt to minimize the impact of human error in a process is called: a. Poka-yoke b. Jidohka c. Kaizen d. Muda

a. Poka-yoke Muda means waste. Jidhoka is a form of visual board. Kaizen is a concept meaning continual improvement. Poka-yoke is a means of mistake proofing a process.

Review of purchase orders for quality requirements falls into which one of the following quality cost segments? a. Prevention b. Appraisal c. Internal failure d. External failure

a. Prevention

In developing a chart to plot a course of action, with many of the events or milestones unknown, which new quality management tool would be used? a. Process decision program chart b. Activity network diagram c. Matrix diagram d. Affinity diagram

a. Process decision program chart

The project charter will be useful in many ways, including: a. Providing a consistent target for the team b. Permitting the team leader to develop milestones from it c. Assuring the champion will assign responsible team members d. Ensuring team members will support the charter

a. Providing a consistent target for the team

An advantage of the R&R range method compared to the average and range or ANOVA methods is that it is a quick way to: a. Quantify the total R&R portion of measurement b. Quantify the repeatability portion of measurement c. Quantify the reproducibility portion of measurement d. Quantify any part and technician measurement interactions

a. Quantify the total R&R portion of measurement

Assume in an R&R study, using the ANOVA method, that the technician-to-technician error was noted to be very low. This value could also be stated as low: a. Reproducibility error b. Repeatability error c. Interaction error d. Process error

a. Reproducibility error

In highly effective Six Sigma companies, most employees receive some training. What group is most likely to receive sponsorship training? a. Senior management b. Master black belt candidates c. Black belt candidates d. Green belt candidates

a. Senior management Sponsorship training is normally given to upper management. Master black belt, black belt, and green belt candidates receive more extensive technical instruction.

A relational matrix is a problem-solving tool which helps to: a. Shows cause-effect relationships between input and output variables b. Focus attention on problems in priority order c. Generate a large number of improvement ideas d. Determine where nonconforming parts exist

a. Shows cause-effect relationships between input and output variables

The major purpose of rolled throughput yield (RTY) in the Six Sigma define step would be to: a. Spot significant differences in yield b. Assist in the project management planning c. Use the calculation for customer analysis d. Determine what process steps can be avoided

a. Spot significant differences in yield

Consider the following definition: "The best combination of machines and people working together to produce a product or service at a particular point in time." What lean concept is being described? a. Standard work b. A future state map c. The value stream d. Ultimate cycle time

a. Standard work

In determining a process average fraction defective using inductive or inferential statistics, one would be using: a. Statistics, computed from samples, to make inferences about populations b. Populations, computed from samples, to make inferences about populations c. Samples, computed from statistics, to make inferences about populations d. Samples, computed from populations, to make inferences about samples

a. Statistics, computed from samples, to make inferences about populations

At what stage in the 5S program would visual controls be employed? a. Step 4 standardize b. Step 3 scrub c. Step 2 straighten d. Step 1 sort

a. Step 4 standardize

One would suspect that process FMEA would result from: a. Studying the manufacturing and assembly process b. Interaction between the systems, subsystems and components c. Investigating the function requirements and design alternatives d. Review on failure modes before the first service

a. Studying the manufacturing and assembly process As the answers are presented, answer a best describes a process FMEA. The other answer choices are b (system FMEA), c (design FMEA), and d (service FMEA).

In a continuous flow manufacturing (CFM) line, the time allowed for each line operation is called the: a. Sequence time b. Takt time c. Lapse time d. Delay time

a. Takt time

Effective communications ensure which of the following? a. That business objectives are understood b. That the strategic plan is established c. That business success is assured d. That customer requirements have been neglected

a. That business objectives are understood

Identify the assumption that is made when conducting an experiment: a. That the measurement system is capable for all included responses b. That the selected factors are the only ones of importance c. That the process changes radically during the duration of the test d. That residuals are not important

a. That the measurement system is capable for all included responses

The basic statistical principle in EVOP is: a. The ability to recognize small differences through large sample sizes b. Operating with low levels of confidence c. Making large changes in independent variables d. Determining dome contours

a. The ability to recognize small differences through large sample sizes

Which of the new quality management tools is used to organize facts and data about an unfamiliar subject or problem? a. The affinity diagram b. The header technique c. The activity network diagram d. Matrix diagram

a. The affinity diagram

An improvement team has completed several RPN values and is now ready to institute improvement. There are three failure modes with the same RPN score in the table below. The first line item which should be reduced would be: a. The bad battery because it has the highest severity ranking b. The cracked welds because of the highest occurrence c. The missing bolts because it has the highest detection level d. The rankings are the same, so choose any one to start

a. The bad battery because it has the highest severity ranking AIAG advises that priority is placed on high severity, followed by RPN numbers. The severity category must be addressed first.

Gantt chart advantages include which of the following? a. The charts are easy to understand b. Changes can rarely be made c. Each bar represents multiple activities d. Estimates of optimistic and pessimistic times can be included

a. The charts are easy to understand

Effective team mechanics would typically include which of the following? a. The development of an agenda and distribution of minutes b. The support of upper management c. Problem solving training d. The four classical team development stages

a. The development of an agenda and distribution of minutes

Lessons learned for a project would typically include which of the following? a. How to increase development time for similar projects b. The effectiveness of the entire project c. How to replicate any mistakes that were made d. Increasing resources for the next project

a. The effectiveness of the entire project

During the team building phase, which of the following best describes the actions of the team? a. The group is uncertain of their duties b. Members prioritize and perform tasks c. Member cooperation is evident d. The team leader usually delegates duties

a. The group is uncertain of their duties The question requires some knowledge of team life cycles. Some authorities refer to the team life cycle phases as build, develop, and optimize. Others use forming, storming, norming, and performing. The building phase is an early one in which the leader provides provides more direction and the group is uncertain of their roles and duties (answer a). During this period, the leader does not delegate responsibilities readily and the team usually does not have the skills and experience to prioritize and perform tasks without assistance.

The most difficult type of variability to reduce via control charting and/or other techniques is? a. The inherent error of measurement b. Piece to piece variation c. Stream to stream variability d. Lot to lot variability

a. The inherent error of measurement

To state that a model in an experimental design is fixed indicates that: a. The levels used for each factor are the only ones of interest b. The levels were chosen from a fixed population c. The equipment from which the data are collected must not be moved d. The factors under consideration are qualitative

a. The levels used for each factor are the only ones of interest

Consider the following data set: 3, 4, 7, 7, 8, 11, 12, 13. Which of the following are factual statements regarding this data? a. The mean is greater than the mode b. The median is less than the mode c. The mean is less than the median d. The mode and median are the same

a. The mean is greater than the mode

A combined calculation of repeatability and reproducibility using the average and range method produces a ratio of 7.42% of process tolerance. What can be stated about the 7.42% value? a. The measurement system is acceptable b. The measurement system is marginal c. The measurement system is not acceptable d. It must be separated into individual R&R values

a. The measurement system is not acceptable

One could say that the underlying objective of TPM is: a. The near elimination of the six big equipment effectiveness losses b. Operating equipment at or near their designated speeds c. Permitting no setup or adjustment losses d. Producing minimal scrap or quality defects

a. The near elimination of the six big equipment effectiveness losses

If Cpk (upper) was determined to be 2.0 and Cpk (lower) was determined to be 1.0, what factual statements can be made about the process? a. The process is shifted to the left b. A calculation error has been made c. The process is not stable d. Cpk must be reported as 2.0

a. The process is shifted to the left

In a simple two variable linear regression study, what does the term SS1 represent? a. The slope of the line b. The x axis intercept c. The measurement of interaction d. The y axis intercept

a. The slope of the line

In order to calculate a performance index, what two facts must be known about the process? a. The specification limits and the standard deviation b. The process average and process control spread c. The process standard deviation and Z value d. The process confidence interval and process average

a. The specification limits and the standard deviation

A test of significance using a given value of a is performed on the yield data from a process using a standard material and a proposed substitute. Which of the following conclusions is possible from this test? a. The standard material is better than the substitute material b. There is an interaction between the two materials c. The probability of a type I error is beta d. The sample size is too large to detect any material differences

a. The standard material is better than the substitute material

The iterative approach to DOE refers to: a. The use of sequential experimentation b. Assuring the stability of the process during experimentation c. Assuring the capability of the measurement system d. Appropriate estimates of experimental error

a. The use of sequential experimentation

Random selection of a sample: a. Theoretically means that each item in the lot had an equal chance to be selected b. Ensures that the sample average will equal the population average c. Means that a table of random numbers was used to dictate the selection d. Is a meaningless theoretical requirement

a. Theoretically means that each item in the lot had an equal chance to be selected

For the nominal probability distribution, the relationships among the median, mean, and mode are: a. They are all equal to the same value b. The mean and mode have the same value but the median is different c. Each has a value different from the other two d. The mean and median are the same but the mode is different

a. They are all equal to the same value

The results of lean principles/thinking transformation show the greatest reduction in the areas of: a. Throughput times and inventory b. Customer errors and in-house scrap c. Safety injuries and capital investment d. Productivity and injury rates

a. Throughput times and inventory Actually there is an increase (not a reduction) in productivity. Typically, throughput times and inventories show a 90% reduction. Items like customer errors, in-house scrap, injuries, and product development times are reduced in the vicinity of 50%.

TPM in relationship to the lean enterprise system means: a. Total productive maintenance to maximize equipment effectiveness b. Total preventative maintenance to reduce total cost c. Total process management to manage process cost d. Total preventative maintenance to minimize downtime

a. Total productive maintenance to maximize equipment effectiveness

With regard to conditions on the work floor, which of the following should management consider to be the most valuable information source? a. Upward flow (from below) b. Downward flow (from the top) c. Horizontal flow d. Informal network

a. Upward flow (from below)

A stamping press makes four parts at a time. To set up control charting for this process, the best rational subgrouping should be to: a. Use separate control charts for each activity, using four sequential parts b. Use on control chart for the press, using four parts from one stamping c. Use one control chart for the press, using four sequential parts from one cavity d. Use one control chart for the press with a sample size of six

a. Use separate control charts for each activity, using four sequential parts

There are a large number of potential human errors. What are possible countermeasures for inexperience? a. Visual aids and work instructions b. Education and/or discipline c. Work standardization and discipline d. TPM and skill building

a. Visual aids and work instructions

The difference between setting alpha equal to 0.05 and alpha equal to 0.01 in hypothesis testing is: a. With alpha equal to 0.05, one is more willing to risk a type I error b. With alpha equal to 0.05, one is more willing to risk a type II error c. Alpha equal to 0.05 is a more conservative test of the null hypothesis d. With alpha equal to 0.05, one is less willing to risk either type I or type II error

a. With alpha equal to 0.05, one is more willing to risk a type I error

Determine the 95% confidence interval for a population proportion if 6 defectives were found in a sample size of 100 units? a. 0.021 </= p </= 0.099 b. 0.0135 </= p </= 0.1065 c. 0.011 </= p </= 0.109 d. 0.0245 </= p </= 0.0955

b. 0.0135 </= p </= 0.1065

On average, a company hired four people per month. In a given month, what is the probability that exactly seven people will be hired? a. 0.0003 b. 0.0595 c. 0.4487 d. 0.0087

b. 0.0595

A process consists of three sequential steps with the following yields. Determine the total defects per unit. a. 0.063 b. 0.065 c. 0.067 d. 0.069

b. 0.065

Calculate the standard deviation of the population for the following set of five sample observations: 1.5, 1.2, 1.1, 1.0, 1.6 a. 1.280 b. 0.259 c. 0.231 d. 0.518

b. 0.259

A study was conducted on the relationship between the speed of different cars and their gasoline mileage. The correlation coefficient was found to be 0.35. Later, it was discovered that there was a defect in the speedometers and they had all been set 5 miles per hour too fast. The correlation coefficient was computed using correlated scores. Its new value will be: a. 0.30 b. 0.35 c. 0.40 d. -0.35

b. 0.35

If the probability of a car starting on a cold morning is 0.6, and we have two such cars, what is the probability of at least one of the cars starting on a cold morning? a. 0.84 b. 0.81 c. 0.60 d. 0.36

b. 0.81

The reported Cpk for a process with an average of 28, a spread of 10 units, and lower and upper specification limits of 15 and 35 respectively, would be: a. 1.6 b. 1.4 c. 1.8 d. 0.714

b. 1.4

How many individual data values are considered sufficient to accurately calculate the upper and lower control limits for X-bar - R control charts? a. 20 b. 100 c. 200 d. 500

b. 100

Suppose that five bad electron tubes were mixed with eight good tubes. If two tubes are drawn simultaneously, what is the probability that both are good? a. 8/13 b. 14/39 c. 7/12 d. 36/91

b. 14/39

The critical value for t, when making a two-tailed paired t test, with a sample size of 13, and alpha = 0.05, is: a. 1.782 b. 2.179 c. 2.064 d. 1.711

b. 2.179

A Six Sigma green belt is performing a hypothesis test of two sample means. Sixteen samples of method A and sixteen samples of method B are produced. The standard deviations are unknown, but thought to be the same. How many degrees of freedom are to be used for the t test? a. 16 b. 30 c. 31 d. 32

b. 30

In a t test, alpha is 0.05, therefore: a. 5% of the time we will say that there is no real difference, but in reality there is a difference b. 5% of the time we will say that there is a real difference when there really is not a difference c. 95% of the time, we will make an incorrect inference d. 95% of the time, the null hypothesis will be correct

b. 5% of the time we will say that there is a real difference when there really is not a difference

The current process produces fifty units per shift, a new process yielded fifty-two units per shift for sixteen straight shifts, with a standard deviation of four units per shifts. What is out level of confidence that the process has changed? a. <90% b. 90-90% c. 95-99% d. >99%

b. 90-95%

When assessing the probability of failure for a product or system component, an index value of 9 is assigned. This means: a. That the severity of failure is high b. A high probability of failure c. That the control effectiveness is very good d. That the overall RPN value will be huge

b. A high probability of failure The question addresses the P term in the RPN (risk priority number). A high number indicates a high probability of failure. Answers a and c describe the severity and control effectiveness components of the RPN. If the S (severity) and D (effectiveness) components are fairly low, the RPN value could still be relatively low even with a high probability value.

The simplification of paperwork, minimization of WIP, and reduction of finished goods inventories can be achieved by utilizing: a. Total productive maintenance b. A kanban pull system c. The visual factory d. Appropriate operator training records

b. A kanban pull system

In the manufacture of airplane fuselage frame selections, thousands of rivets are used to join aluminum sheets and frames. A study of the number of oversized rivet holes and the number of minor repairs to a unit yielded a correlation coefficient of +/- 1.08. This means that a. The number of oversized rivet holes on a unit is a good predictor of minor repairs b. A new statistician should be hired c. The number of oversized rivet holes is a poor predictor of minor repairs d. A small number of minor repairs will have to be made

b. A new statistician should be hired

As an alternative to QFD, the plotting of items of customer importance versus the customer's corresponding satisfaction level, using a form of Likert scale would most appropriately be called: a. A matric diagram b. A perceptual map c. A cause-and-effect matrix d. A correlation chart

b. A perceptual map

A process has been experiencing problems lately. The operators charting the process have identified the cause to be due to a change in incoming materials. The problem is: a. Attributed to purchasing b. A special cause c. A common cause d. A normal event

b. A special cause

For organizing information, facts or data into a systematic, logical manner, which of the following new quality tools would be used? a. An interrelationship diagraph b. A tree diagram c. An activity network diagram d. Prioritization matrix

b. A tree diagram

If an operator is presented with a tray containing all parts to be used in the assembly of a product, what does the tray represent? a. A mechanism to convey a small amount of work b. A visual checklist c. A means to speed up the operation d. A control over downtime that might occur later

b. A visual checklist

The Kano model is used to: a. Measure supplier performance b. Analyze customer requirements c. Describe takt time d. Calculate rolled throughput time

b. Analyze customer requirements

The distribution of a characteristic is negatively skewed. The sampling distribution of the mean for large samples, taken from this same distribution, is: a. Negatively skewed b. Approximately normal c. Positively skewed d. Lognormal

b. Approximately normal

An improvement team will make inferences about the relevant importance and sequence of events when using the interrelationship diagraph by the use of: a. Stick notes on a wall b. Arrows leading to or away from a topic c. The start and stopping of a process d. Numeric scores or indices

b. Arrows leading to or away from a topic

If no correlation exists between two variables, then: a. The correlation coefficient should equal a negative value b. As one variable changes, one cannot predict a value for the other variable c. Both variables will decrease simultaneously d. Analysis of variance must be used to determine if an interaction is present

b. As one variable changes, one cannot predict a value for the other variable

In reference to the figure below, which of the following is true? a. A is a subset of B b. B and C are mutually exclusive c. C is a subset of B d. A and C are mutually exclusive

b. B and C are mutually exclusive

Customer expectations follow which hierarchy of needs, from low to high? a. Expected, basic, unanticipated, appreciated b. Basic, expected, desired, unanticipated c. Unanticipated, appreciated, desired, expected d. Desired, expected, appreciated, unanticipated

b. Basic, expected, desired, unanticipated

Why is Six Sigma called TQM on steroids? a. Because of the extensive training element required b. Because of the inclusion of statistical and lean tools c. Because of the heavy impact of top management support d. Because of the impact of cost savings on the bottom line

b. Because of the inclusion of statistical and lean tools The steroid quote comes from the inclusion of statistical items such as DOE, DFSS, statistical analysis, and certain lean manufacturing tools.

As the scatter of points around a regression line becomes greater, r^2 will: a. Be unaffected b. Become smaller c. Become larger d. Approach a value of 1

b. Become smaller

When constructing a power of test curve, one would not be surprised to discover that as alpha (a) increases: a. The value of mu becomes greater b. Beta decreases c. The probability of rejecting the null hypothesis decreases d. The sample size becomes larger

b. Beta decreases

Identify the best way to error proof activities: a. By corrective actions b. By preventive actions c. By containment actions d. By temporary actions

b. By preventive actions

When considering a quality characteristic for process capability calculations, a green belt should consider which of the following? a. Choose those characteristics with the highest process capability index ratios b. Choose a small number of customer defined CTQ requirements c. Choose only normal characteristics to comply with the normality assumption d. Choose all characteristics defined in the procedures and work instructions

b. Choose a small number of customer defined CTQ requirements

The right hand side of a completed "house of quality" (HOQ) displays rankings and values for: a. Customer needs or desires b. Competitive assessments or comparisons c. Design feature measurements and importance d. Design feature interactions

b. Competitive assessments or comparisons The subroof of a HOQ displays design features and the roof shows design feature interactions (d eliminated). The "basement" of a HOQ shows target values, or design features and technical importance (c eliminated). The left side of the HOQ details key customer needs or desires (a eliminated). The right side of the HOQ shows competitive assessments and/or comparisons.

The composition of a team for a typical Six Sigma project should: a. Be composed of interested floor operators and area staff b. Consist of qualified people with the expertise needed c. Consist of a cross-functional blend of people from various departments d. Consist of green belts, at the very least

b. Consist of qualified people with the expertise needed

A p-value is used in many statistical calculations. It can be described as: a. Being similar to the critical statistical value as found in the statistical tables b. Containing an extreme test statistic probability value as obtained from the sample data c. Having the typical set values of 5% and 1% d. Being reported only when significant

b. Containing an extreme test statistic probability value as obtained from the sample data

When a process is not centered relative to specifications, which of the following statements is true? a. Cpk is equal to Cp when the population is normal b. Cpk is the smallest value of either Cpk upper or Cpk lower c. Cpk is the largest value of either Cpk upper of Cpk lower d. Cpk is the value closest to 1 in the partial capability indexes capability

b. Cpk is the smallest value of either Cpk upper of Cpk lower

If a metrics format were being developed to track marketplace response, which of the following items would be included? a. Cost of quality b. Customer retention c. Cycle time reduction d. Profit margin on sales

b. Customer retention Answer a is a measure of resource allocation. Answer c may be an internal resource or cost measurement. Answer d is a measure of profit.

Why is DFSS called the future of Six Sigma? a. There are few remaining DMAIC projects available in most companies b. DFSS roadmaps are more attractive than DMAIC tools c. DFSS better serves the current innovations initiatives of of many companies d. DMAIC projects do not achieve Six Sigma levels without DFSS

b. DFSS roadmaps are more attractive than DMAIC tools Recent publications in both the private and governmental sectors express the need for whole strategies based on systematic innovation. DFSS roadmaps have the potential to become the center of such innovation initiatives. Answer a is not true. There are plenty of DMAIC projects available for most companies. Answer b may be true for certain designers and engineers, but is not true for many employees. Answer d is also not true, many DMAIC projects continue to achieve the desired Six Sigma levels.

Six Sigma project methodology normally begins with what initial step? a. Problem definition b. Define c. Project charter d. Champion approval

b. Define

A process shows lack of stability but yesterday's capability index was so great (1.77) that your supervisor decides to use it as a benchmark for all future process capabilities. What should your advise your supervisor? a. Use the 1.77 value, it is so good that the process instability is insignificant b. Do not use the 1.77 value, first get the process to statistical stability c. Use the 1.77 value, but improve the process until it shows statistical stability d. Do not use the 1.77 value, the unstable process may provide a better index later

b. Do not use the 1.77 value, first get the process to statistical stability

The basic objective of visual factory techniques is to: a. Utilize production and schedule boards b. Enable everyone to maintain contact with the workplace c. Regulate production using kanban cards d. Make problems visible using tool boards and jidhoka devices

b. Enable everyone to maintain contact with the workplace

When P(A|B) does not equal P(A), then: a. Events A and B are independent b. Events A and B are dependent c. Events A and B are mutually exclusive d. Events A and B are components

b. Events A and B are dependent

The calculation for reproducibility using the average and range method comes from: a. Examining the variation between the appraisers and within their readings b. Examining the variation between the average of the appraisers for all parts measured c. Checking the variation within each appraiser group d. Examining the variation between the least two separate instruments

b. Examining the variation between the average of the appraisers for all parts measured

Identify the most difficult limitation in achieving continuous flow in manufacturing environment: a. Untrained employees b. Existing equipment c. Employee culture d. Middle management involvement

b. Existing equipment All answer choices except existing equipment can be changed by training and management improvement to obtain continuous flow. However, replacement of existing equipment requires capital, management support, and planning.

All of the factors that might be contributing to a production problem must be discovered. Which of the following problem-solving tools would be the best selection? a. Pareto diagrams b. Fishbone diagrams c. Histograms d. Control charts

b. Fishbone diagrams

Pareto diagrams are used to: a. Determine a cause-effect relationship between one or more variables b. Focus attention on problems in priority order c. Generate a large number of ideas d. Display causes in a nongraphical manner

b. Focus attention on problems in priority order

A project team has been functioning very well for about two months. One member has suggested a different approach to solving the problem they have been working on. The other members strongly reject the idea. This is an example of: a. Wanderlust b. Groupthink c. Floundering d. Active-passive behavior

b. Groupthink

When selecting a project, priority should first be given to a project that: a. Only affects employees in the work cell b. Has objectives that align with organizational goals c. Is expected to be completed within one week d. The solution is readily apparent before the project is started

b. Has objectives that align with organizational goals

The fundamental purpose of establishing teams is to: a. Provide team members a form of job enrichment and broadening b. Improve the internal efficiencies of the organization c. Teach team members new problem solving skills d. Avoid spending money on outside consultants

b. Improve the internal efficiencies of the organization

Reduced cycle times for a product can result in reduced work in progress, reduced waste, and: a. Increased production costs b. Improved operations c. Excess inventories d. Longer takt times

b. Improved operations

Which of the following statements is true about a Latin square design? a. It minimizes the chance for factor confounding b. It does not allow for the effects of interaction in the design c. It eliminates the need to ensure that the effects of interaction are additive d. It is useful because the underlying distribution does not need to be normal

b. It does not allow for the effects of interaction in the design

A process FMEA is generally performed: a. Just after the design FMEA is completed b. Just before the production tooling is authorized c. Just after the design drawings are finalized d. As soon as a manufacturing defect is uncovered

b. Just before the production tooling is authorized The process FMEA is initiated before or after the feasibility stage, prior to production tooling, and accounts for all manufacturing operations. Answers a and c are partially correct, in that, PFMEA is usually started after the DFMEA, but it could be some time before production activities start up. Answer d should lead the process owner to review the FMEA to check if another failure mode has occurred. A PFMEA should be in place before manufacturing commences.

The visual factory would be supported to the greatest extent by: a. Muda and gemba b. Kanban and 5S c. Gemba and kanban d. 5S and Muda

b. Kanban and 5S

The coefficients in the equation below can be determined using: Yi=a+bxi+ei a. Inference testing b. Least squares regression c. Sum of squares estimation d. Hypothesis testing

b. Least squares regression

When one works through a conflict situation with a customer: a. There is no conflict, the customer is always right b. The process should not embarrass either party c. The company should maintain its position d. All steps and procedures are strictly followed

b. The process should not embarrass either party

Consider the SS and MS columns of an analysis of variance table for a single factor design. The appropriate ratio for testing the null hypothesis of no treatment effect is: a. SS treatments divided by SS residual b. MS treatments divided by MS residual c. SS treatments divided by MS residual d. MS treatments divided by SS residual

b. MS treatments divided by MS residual

An advantage of training managers in Six Sigma concepts before improvement team members is which of the following? a. Managers have more time available for training b. Managers must lead the deployment of Six Sigma c. Managers must understand the concepts better than the team members d. Managers must pay for worker training

b. Managers must lead the deployment of Six Sigma Managers normally have as much as (or more) time pressure as the other employees. Thus, answer a is untrue. Managers must lead the effort, validate the training, and understand the quality concepts. They don't necessarily need to understand the concepts better than the team members. Answer d may be true, but it fails to address the question.

The essence of kanban concepts includes which of the following? a. Delivery of components and products ahead of need b. Minimal storage in production areas c. Minimal distress in the production system when a machine failure occurs d. Wide applicability to repetitive and non-repetitive production plants

b. Minimal storage in production areas

Most fire hydrants in the US have a pentagon shaped head on the valve stem. The primary reason is that the head is not a square of hexagon is: a. To encourage the sale of the specialized wrenches for operation b. Mistake proofing so only proper authorities can turn on the water c. From a patent by Frederick Graff in 1801 in Philadelphia d. Indicating opening the valve is a clockwise rotation

b. Mistake proofing so that only proper authorities can turn on the water

Overproduction, scrap, waiting, and excess motion are all forms of: a. TPM b. Muda c. Kanban d. CFM

b. Muda

Of the various statistical analysis tools available, which on would be the most likely to show a plot of all reading taken? a. X-bar / R charts b. Multi-vari charts c. ANOVA d. Chi-square

b. Multi-vari charts

If events cannot occur simultaneously they are called: a. Randomly selected b. Mutually exclusive c. Independent d. Statistically stable

b. Mutually exclusive

Determine whether the following two types of rockets have significantly different variances at the 5% level. Assume that the larger variance goes in the numerator. a. Significant difference because Fcalc < F table b. No significant difference because Fcalc < F table c. Significant difference because Fcalc > F table d. No significant difference because Fcalc > F table

b. No significant difference because Fcalc < F table

Techniques useful for team facilitators when narrowing a list of potential problem areas to investigate include which of the following? a. Brainstorming and recording ideas b. Nominal group technique and multivoting c. Voting and brainstorming d. Multivoting and idea incubation

b. Nominal group technique and multivoting

The normality of residuals for factor A indicate a uniform behavior around zero, what can be concluded from this fact? a. There is something wrong with the model. Go back to the planning stage b. Normality of the residuals has been demonstrated by the uniform behavior. c. Factor A is a nuisance factor, as been demonstrated by the uniform behavior. d. Factor A is statistically significant because of the uniformity of residuals.

b. Normality of the residuals has been demonstrated by the uniform behavior

Which of the following statements describes discrete data? a. It takes 3 hours and 48 minutes to fly from LA to New York b. Of 225 people on the airplane, 85 had connecting flights c. The flight arrived at 9:08pm d. There were 5,923 gallons of fuel consumed on the flight

b. Of the 225 people on the airplane, 85 had connecting flights

A null hypothesis states that a process has not improved as a result of some modifications. The type II error is to conclude that: a. One has failed to reject the null hypothesis when it was true b. One has failed to reject the null hypothesis when it was false c. One has rejected the null hypothesis d. One has made a correct decision with an alpha probability

b. One has failed to reject the null hypothesis when it was false

Which statement best describes an optimized risk management process? a. Cost of resolution equals risk exposure b. Overall cost and risk are minimized c. Risk mitigation plans that are rarely exercised d. A saleable and flexible plan

b. Overall cost and risk are minimized

For two events, A and B, which of the following is a true probability statement? a. P(A or B) = P(A) + P(B) if A and B are independent b. P(A or B) = P(A) + P(B) if A and B are mutually exclusive c. P(A and B) = P(A) x P(B) if A and B are mutually exclusive d. P(A or B) = P(A) x P(B) if A and B are independent

b. P(A or B) = P(A) + P(B) if A and B are mutually exclusive

Which of the following quality management tools use techniques that incorporate a form of flow charting? a. Tree diagrams, activity network diagrams and matrix diagrams b. PDPC charts, tree diagrams, activity network diagrams c. Affinity diagrams, tree diagrams, interrelationship diagrams d. Matrix diagrams, PDPC charts, tree diagrams

b. PDPC charts, tree diagrams, activity network diagrams

After the implementation of TPM, what is the long term objective? a. Accessibility b. Perfection c. Modularization d. Standardization

b. Perfection

A Six Sigma green belt has run a 2^5-2 fractional factorial design. Only two of the five factors remained important after the factorial analysis. Escalation of this DOE to an optimization model will be best accomplished by: a. Performing a new full factorial of the two factors b. Performing a response surface experiment c. Decreasing the degrees of freedom of the original DOE d. Adding an additional factor level

b. Performing a response surface experiment

Identify the quality guru who believed the best approach to understanding the purpose of a quality system would be the four absolutes of management: a. Dr. Feigenbaum b. Philip Crosby c. Dr. Deming d. Dr. Juran

b. Phillip Crosby The four absolutes of quality management are part of Phillip Crosby's message.

Given a coefficient of determination of 0.9, what is the correlation coefficient? a. 0.9 or -0.9 b. 0.81 or -0.81 c. 0.9487 or -0.9487 d. 0.9487

c. 0.9487 or -0.9487

The term severity in a FMEA describes the: a. Difficulty of completing the FMEA form b. Possible impact to a system user of failure c. Likelihood of a failure d. Time for which the system is expected to be down

b. Possible impact to a system user of failure This question requires a basic knowledge of FMEA terms. The severity is the effect it will have on the system, the operators, or the mission. Answer c is the probability of failure. Answer d is the downtime. Answer a is a distractor choice.

A project has been completed and the final report written. The next activity relevant to the project is: a. Benefit-cost analysis b. Postmortem analysis c. Reward and recognition of project team members d. Document archiving

b. Postmortem analysis

Womack stated that perfection in a lean organization should be: a. Specified b. Pursued c. Identified d. Pulled

b. Pursued All of the answer choices have some connection to Womack's five guiding principles. However, he specifically stated that perfection should be pursued.

Out of Dr. Deming's 14 points for management states "Cease dependence upon inspection as a way to achieve quality." The underlying tenet of this statement is which of the following? a. Many American companies employ too many inspectors; perhaps 5% - 10% of the work force b. Quality should be built into the product, not inspected in c. In most cases, the worker should perform his/her own inspection and not rely on someone else d. Many manual inspections will miss 10% - 20% of defects under typical working conditions

b. Quality should be built into the product, not expected in This question requests the underlying tenet or real reason that inspection dependence should be minimized or eliminated. Answers a, c, and d are true statements in many circumstances. They explain or expand some of the weaknesses or facts present during product inspection. However, answer b explains the question best. If quality is built into the product, then it doesn't matter if inspection misses a defect. The defect isn't there. Inspection to improve quality is too late, ineffective, and costly. Harold F. Dodge said "You cannot inspect quality into a product."

In a production factory, which utilizes the pull system, which of the following may be achieved? a. Expand raw material inventory b. Reduced in-process inventory c. Increased prices d. Increased finished goods inventory

b. Reduced in-process inventory The pull system is driven by customer demand. Hence, inventory will be less. Generally, stabilized prices and increased profits can be achieved. An increased price is not the objective.

A statistical software program returned a p-value of 0.023. If the desired level of significance is 0.025, then the conclusion is: a. Reject the null hypothesis, there is no statistical difference b. Reject the null hypothesis, there is a statistical difference c. Fail to reject the null hypothesis, there is no statistical difference d. Fail to reject the null hypothesis, there is a statistical difference

b. Reject the null hypothesis, there is a statistical difference

Risk planning requires inputs such as: a. Avoidance b. Resources c. Transference d. Mitigation

b. Resources

Correlation coefficients are generated from which of the following graphs? a. Measles charts b. Scatter diagrams c. Pareto diagrams d. Control charts

b. Scatter diagrams

American visitors to an aluminum plant in Japan were given white gloves to wear. What category of the 5S program was being displayed? a. Sort b. Scrub c. Straighten d. Standardize

b. Scrub

If you are reading this question, you are a customer of QCI. Identify a QCI process output element from the list below: a. Binder manufacturers b. Solution texts c. Authors d. Paper companies

b. Solution texts

Which Six Sigma role is most likely to define objectives for an improvement team? a. Leader b. Sponsor c. Facilitator d. Member

b. Sponsor

The most difficult stage for any team to work through is: a. Norming b. Storming c. Performing d. Forming

b. Storming

Review the following 5S elements and identify the step that is being referenced...Determine who has missing items; create a name and location for everything; use aisle and material placement markings; use labels, tool boards, and color codes a. Sort b. Straighten c. Scrub d. Systematize

b. Straighten

At what 5S step would standards for each of the five steps be established a. Sort b. Sustain c. Scrub d. Straighten

b. Sustain

Consider the following multi-vari chart of a single product measured in the same four locations, across width, over time. Evaluating the chart by eye, arrange the categories of variation from largest to smallest. a. Temporal, cyclical, positional b. Temporal, positional, cyclical c. Positional, cyclical, temporal d. Positional, temporal, cyclical

b. Temporal, positional, cyclical

They key attribute for individual team members is: a. That they are able to work schedules to accommodate team activities b. That they believe in the value of the team process c. That they have extensive experience within the organization d. That they understand the full scope of the problem at hand

b. That they believe in the value of the team process

A Six Sigma project team is led by a green belt. The current activity is to develop a control plan. The ultimate responsibility for developing the initial plan would belong to: a. The project team b. The green belt c. The process owner d. The production function

b. The green belt

What process controls the tempo of the value stream? a. The first operation b. The pacemaker process c. The future state map d. The last process

b. The pacemaker process

Because of the large number of variables under study, an engineer is considering a fractional factorial instead of a full factorial to analyze a process. Apart from the possibility of studying a large number of factors with relatively few experiments, what other characteristic will support a decision to use a fractional factorial instead? a. It is suspected that there are many interactions b. The process is well known and only the main factors are of concern c. A fractional factorial will determine the main effects curvature d. Blocking is necessary to account for nuisance factors in this study

b. The process is well known and only the main factors are of concern

Hearing aids and eye glasses are examples of: a. Value stream mapping b. Kanban c. Poka-yoke d. Kaizen

c. Poka-yoke

Consider that a sample of ten units is taken from a population. Which of the following statements is correct? a. The standard deviation is usually greater than the variance b. The range is greater than the standard deviation c. A sigma estimate cannot be determined d. No estimate of the population average is possible

b. The range is greater than the standard deviation

After attending SPC classes, a second shift production supervisor implements a mean chart for an important quality characteristic. The supervisor said "I'm happy to announce that out of 24 sample means (sample size 5 units, taken every 20 minutes) none were found outside of specification limits. The process is running flawlessly." What can be stated about the supervisor's conclusion? a. The supervisor is wrong, there is no measure of the confidence level b. The supervisor is wrong, two different populations are being compared c. The supervisor is right, for the wrong reasons d. The supervisor is right, all the values are within specifications

b. The supervisor is wrong, two different populations are being compared

Why should an experimenter plot data point and graph the least squares line if a probabilistic regression model exists? a. To visually present the relationship to others b. To check for fit; there may be a calculation error c. There is no rational reason for doing so d. To permit a projection outside of the test area

b. To check for fit; there may be a calculation error

Which of the following DOE statements is correct? a. Variables are confounded if they are difficult to study b. Two or more variables are confounded if their effects cannot be separated c. Variables are confounded if they form a linear combination d. Two or more variables are confounded if they produce the same effects

b. Two or more variables are confounded if their effects cannot be separated

The most common subgrouping scheme for X-bar - R control charts is to separate the variation: a. Within stream versus stream-to-stream b. Within time versus time-to-time c. Within piece versus piece-to-piece d. Inherent process versus error of measurement

b. Within time versus time-to-time

As a new green belt in the company, you are given the following data: X-double bar = 4.241 mm, S- x bar = 0.565mm, n=5 You decide to estimate the process parameters but discover that the original data was lost and all you have are these 3 numbers. What is the best estimate that can be made of the process parameter under the current circumstances? a. sample mean = 4.241 and sample std dev = 0.565 b. sample mean = 4.241 and sample std dev = 1.263 c. sample mean = 4.241 and sample std dev = 0.253 d. sample mean = 1.896 and sample std dev = 1.263

b. sample mean = 4.241 and sample std dev = 1.263

Which table should be used to determine a confidence interval on the mean when s is not known and the sample size is 10? a. Z b. t c. F d. X^2

b. t

A regression analysis yielded a total sum of squared errors of 1000 and a total sum of squares equal to 1600. What is the correlation coefficient? a. Cannot be determined from the given information b. +0.375 or -0.375 c. +0.612 or -0.612 d. +0.790 or -0.790

c. +0.612 or -0.612

In most Japanese based companies utilizing TPM, the goal for equipment breakdown losses is: a. 5 minutes or less b. 10 minutes or less c. 0 minutes d. 15/20 minutes

c. 0 minutes

A lot of 50 pieces contains 5 defectives. A sample of two is drawn without replacement. The probability that both will be defective is approximately: a. 0.0100 b. 0.0010 c. 0.0082 d. 0.0093

c. 0.0082

ASQ sectional history indicates that 70% of all candidates successfully pass the CSSGB certification exam. A total of 12 company employees (including your) take the upcoming CSSGB exam. The area manager has promised a big bonus if all 12 you pass the exam. What is the probability of getting promised bonus. a. 0.700000 b. 0.083000 c. 0.013841 d. 0.001176

c. 0.013841

A process is producing material which is 20% defective. Five pieces are selected at random for inspection. What is the probability of exactly three good pieces being found in the sample? a. 0.184 b. 0.061 c. 0.205 d. 0.051

c. 0.205

A p chart has been plotted for some time. Recently, steps have been made to substantially improve the process. One would not be surprised to find that: a. The chart demonstrates more out of control conditions b. The chart must be converted into a variable chart c. A larger sample size must be taken d. The chart requires the samples to be taken more frequently than in the past

c. A larger sample size must be taken

What is a value stream? a. The elimination or reduction of internal waste b. The minimization of internal, hidden muda c. A map identifying all product related activities d. The value as perceived by the customer

c. A map identifying all product related activities All of the answer choices have some connection to value and/or value stream. Specifically, however, a value stream is a map of all product related activities.

The term "variables" can be described in the following way: a. A quality which can be absent or present in a product b. A definable attribute or characteristic of a product c. A quality which can assume several (more than two) values d. A quality which is absent in a product in one or more specifications

c. A quality which can assume several (more than two) values

Which control chart pattern best represents an in control process? a. A consecutive run of seven or more points on one side of the centerline b. A random distribution of points with one point outside of the control limits c. A random distribution of points on both sides of the centerline d. A steady trend of points toward either control limit

c. A random distribution of points on both sides of the centerline

The project charter will contain a business case, which can be defined as: a. A reasoning for the redesign of a process or product b. The full arguments for the project c. A short summary of the strategic reason for the project d. A case study of the project situation

c. A short summary of the strategic reason for the project

A number resulting from the manipulation of some raw data according to certain specified procedures is called: a. A population b. A constant c. A statistic d. A parameter

c. A statistic

One-way analysis of variance is most similar in its objectives to: a. A test of population mean b. A test for equality of two sample proportions c. A test for equality of two population means d. A chi-square test for independence

c. A test for equality of two population means

Identify the data conversion that would be most difficult to accomplish: a. Attribute data converted to variables data b. Variables data converted to attribute data c. Accumulated "go/no-go" data converted to variables data d. Variables data converted to "go/no-go" data

c. Accumulated "go/no-go" data converted to variables date

It is often desirable to do an R&R study of a gage system and compare it to the total process variation. What must be assured about the gage being used before an R&R study can be performed? a. Accuracy and precision b. Reliability and sensitivity c. Accuracy and sensitivity d. Reliability and precision

c. Accuracy and sensitivity

A designed experiment of three factors (A,B, and C) at two levels was conducted. The eight runs were analyzed, suggesting that one level of factor A showed significant improvement. The plant manager stated that no additional runs are needed. The best response is: a. The experiment did its job and should be closed b. The results of the test should be implemented immediately c. Additional replications are needed to verify the experiment d. An EVOP should be conducted to improve the process further

c. Additional replications are needed to verify the experiment

Plackett and Burman experimental designed are called screening designs. A screening design can be defined as: a. An experiment with interactions among the main effects b. The use of non-geometric experimental designs c. An identification of the key input factors d. A fractional factorial experiment

c. An identification of the key input factors

Parallel experiments are conducted: a. At two or more times b. In sequence c. At the same time d. By a single person doing multiple treatments

c. At the same time

What tool is customarily used to complement the cause-and-effect diagram? a. Scatter diagrams b. Pareto diagrams c. Brainstorming d. Force field analysis

c. Brainstorming

Understanding, controlling, and improving an organization's processes to create value for all stakeholder's would be called: a. The SIPOC diagram b. Process performance metric c. Business process management d. The establishment of KPIVs and KPOVs

c. Business process management

Which of the following distributions have their x axis starting at 0? a. Normal and t b. Normal and chi-square c. Chi-square and F d. F and t

c. Chi-square and F

If a team leader wishes to take an assertive stance in dealing with a conflict, he or she will want to select which of the following positions? a. Avoiding b. Accommodating c. Collaborating d. Comprising

c. Collaborating

Which of the following statements is true? a. Confidence intervals increase in width as the sample size increases b. Confidence intervals are always symmetrical c. Confidence intervals for the mean are independent of the population distribution d. Confidence intervals are independent of the sample size

c. Confidence intervals for the mean are independent of the population distribution

When making measurements with test instruments, precision and accuracy mean? a. The same b. The opposite c. Consistency and correctness, respectively d. Exactness and traceability, respectively

c. Consistency and correctness, respectively

The concept behind PDCA is: a. The Deming/Shewhart cycle b. Process flow c. Continuous improvement d. Satisfying suppliers

c. Continuous improvement The key question phrase is "concept behind." Answer b is inappropriate and does not fit the question. Answer a would be correct if the question requested another name for PDCA. Answer d could be one of a number of potential positive outcomes of this activity. However, the concept behind and objective of PDCA is one of continuous improvement.

A graphical display of the total percentage of results below a certain measurement value is called a: a. Histogram b. Probability density function c. Cumulative distribution function d. Expected value

c. Cumulative distribution function

The analysis of risk involves two measures of failure. These measures are: a. Failure analysis and failure effects b. Failure mode and failure method c. Failure severity and failure probability d. Failure mechanism and failure mode

c. Failure severity and failure probability This question requires familiarity with reliability risk assessment. The two key measures of risk assessment are the severity and probability of failure.

The advantage of using the modern designed method of experimentation, rather than the classical, is that: a. Everything is held constant except for the factor under investigation b. Experimental error is recognized but need not be stated in quantitative terms c. Fewer terms and measurements are needed for valid and useful information d. The sequence of measurement is often assumed to have no effect

c. Fewer terms and measurements are needed for valid and useful information

The expression below is which of the following? n!/x!(n - x)! p^x(1 - p)^n-x a. General term for the Poisson distribution b. General term for the normal distribution c. General term for the binomial distribution d. General term for the chi-square distribution

c. General term for the binomial distribution

When two team members express strong opposite views, the remaining team members should: a. Ask both members to leave b. Vote that the leader stop the discussion c. Have the two clearly state their positions d. Use the sponsor to clarify the situation

c. Have the two clearly state their positions.

Process mapping of activities and systems is most helpful in detecting: a. Ways to eliminate written procedures b. Deficiencies in the organizational structure c. Holes or gaps in the control system d. Improper use of statistical methods

c. Holes or gaps in the control system

Understanding customer needs is a constant requirement for organizations. Customer data and information is often collected in various amounts. The organization should resolve to: a. Use more reactive approaches b. Eliminate process redundancy c. Identify customer satisfaction drivers d. Sort out the unneeded customer data

c. Identify customer satisfaction drivers

The risk priority number (RPN) is the measure of risk related to a particular failure mode. The ratings for each category (severity, occurrence, and detection) are on a 1 to 10 scale. If the FMEA has 10 failure modes with RPN numbers ranging from 50 to 300, an improvement team's recommended actions should focus on which of the following? a. Work on those failure modes with RPNs greater than 50 b. Use the Pareto principle to prioritize the highest RPNs c. Identify failure modes with high severity rankings, followed by high RPN numbers d. Use the Pareto principle and start with the top 20% of the failure modes

c. Identify failure modes with high severity rankings, followed by high RPN numbers Answer a is incomplete. One must go beyond initial RPN. Answer b is a good starting point, but answer c points out the need to reduce those high severity rankings first, and then look at those with high RPNs. Answer d is an incomplete choice, since one must also consider severity numbers.

In the regression equation y = mx + b, y increases with x in all cases: a. If b is positive b. If b is negative c. If m is positive d. If m is negative

c. If m is positive

Identification of the external customer is important because: a. It eliminates wasted advertising b. It produces more profit per customer c. It helps identify the customer's needs d. It permits easier product recalls

c. It helps identify the customer's needs

Which of the following quality luminaries would most clearly identified as a proponent of improvement and breakthrough project? a. Ishikawa b. Deming c. Juran d. Crosby

c. Juran Juran's Trilogy a consists of three processes: quality planning, quality control, and quality improvement. The quality improvement phase would certainly be considered a project approach to improvement.

Which of the following is a single factor experiment containing two specific nuisance factors? a. Graeco-Latin square b. Fractional factorial experiment c. Latin square d. Mixture design

c. Latin square

What primary activities take place during most team meetings? a. Conflict between the facilitator and team members b. Taking detailed minutes and publishing information promptly c. Learning the teamwork process and improving the work process d. Consensus building and conflict resolution

c. Learning the teamwork process and improving the work process

Which of the following data types would provide the most useful information in eliminating the causes of paint blemishes on automobiles being produced on an assembly line? a. Attribute b. Ordinal c. Locational d. Variables

c. Locational

The team's charter describes the team's a. Leader, facilitator, recorder, and timekeeper b. Meeting dates, milestones, and targets c. Mission, scope, and objectives d. Members, sponsors and facilitators

c. Mission, scope, and objectives

On an X-bar chart, many plot points are noted in zone B, some in zone C, and few in zone A. What could be occurring in the process? a. Trends or over corrections b. Stratification c. Mixtures or systematic variation d. Improper calculation of the control limits

c. Mixtures or systematic variation

When calculating a sample size for a DOE, one is actually calculating the: a. Number of runs b. Number of factors c. Number of replications d. Number of blocks

c. Number of replications

If events A and B are independent, then: a. P(B/A) = P(A) b. P(A/B) = P(B) c. P(B/A) = P(B) d. P(B/A) = P(A) + P(B)

c. P(B/A) = P(B)

What is the practical difference between the precise/tolerance ratio (P/T) ratio and the precision/total variation ratio (P/TV ratio). a. P/T ratio and P/TV ratio are practically the same b. P/T ratio give a better picture of the measurement precision for internal improvement studies while P/TV ratio is better for evaluations relative to specifications c. P/T ratio gives a better picture of the measurement precision relative to specifications while the P/TV ratio is better for internal improvement studies d. The combined P/T and P/TV ratios should equal one in order to achieve a better understanding of the measurement system analysis results

c. P/T ratio gives a better picture of the measurement precision relative to specifications while the P/TV ratio is better for internal improvement studies

In most cases, an improvement team receives the least control and direction during which of the following stages a. Building b. Storming c. Performing d. Alarming

c. Performing

In what areas would upper management be most helpful in the initiation of a Six Sigma effort? a. Providing direct training to black belts b. Standardizing business operations c. Providing key resources to the organization d. Directing the improvement projects

c. Providing key resources to the organization In Six Sigma, as in more other strategic initiatives, upper management functions best by providing overall direction and resources. Upper management probably doesn't have the skills to provide black belt training and should not personally direct the improvement projects. Standardizing businesses operations makes sense only if they are at the best level possible.

Which of the following measures of variability is independent of the exact value of every measurement? a. Mean deviation b. Variance c. Range d. Standard deviation

c. Range

In a visual factory setting, kanban cards are typically used to: a. Identify areas needing housekeeping attention b. Clarify targets for future improvement c. Regulate the flow of production and inventory d. Make production problems visible

c. Regulate the flow of production and inventory

Identify a non-value added activity that would be considered a form of processing muda: a. Any and all forms of transportation b. An ergonomically unsound workplace c. Reshaping a product due to poor dies d. Producing more than needed by the customer

c. Reshaping a product due to poor dies This question requires some detailed knowledge of muda categories: answer a is transportation muda, answer b is motion muda, and answer d is overproduction.

As a good experimenter, you have built a predictive model of the experimental data. The differences between the actual response data and the model data are termed: a. Confounded data b. Nested experiments c. Residuals d. Efficiency of estimators

c. Residuals

What is the main difference between risk analysis and risk management? a. There is minimal difference, they refer to the same concept b. Risk analysis refers to tools and risk management deals with consent c. Risk analysis evaluates risk while risk management is a more inclusive process d. Risk analysis includes risk handling while risk management refers to risk monitoring

c. Risk analysis evaluates risk while risk management is a more inclusive process

White glove inspections would most likely occur during which 5S stage? a. Sort b. Straighten c. Scrub d. Sustain

c. Scrub

As with the seven quality management tools, the Japanese concept of workplace organization, 5S, has been Americanized. Of the five original tools, which is the hardest to find in the American system? a. Sieri b. Seiton c. Shitsuke d. Seiso

c. Shitsuke

The hybrid option on a $25,000 car costs $3,000. A gas mileage tests found the hybrid averaged 39.1 mpg and the standard model averaged 34.7 mpg and a p-value of 0.024. At a level of significance of 5%, the difference is: a. Not statistically significant, buy the standard car b. Not practically significant, buy the hybrid car c. Statistically significant, but not practically significant d. Statistically significant, one should buy the hybrid car

c. Statistically significant, but not practically significant

Which of the following statistical term statements is correct? a. Parameters come from samples b. Samples come from statistics c. Statistics come from samples d. Populations come from statistics

c. Statistics come from samples

If an engineer or technician were to select samples from a mixture in a vat that is suspected of separation, what sampling technique would be advisable? a. Random sampling b. Sequence sampling c. Stratified sampling d. Discovery sampling

c. Stratified sampling

Short-term tactical plans are: a. Usually defined by the customer b. Set before objectives can be determined c. Supportive of strategic objectives d. Normally opposite of long range objectives

c. Supportive of strategic objectives Short-term tactical plans or operational plans are used to implement long and short-term strategic plans. Functional departments develop short-term objectives, budgets, policies, and programs to support the organization's strategic plans. Short-term plans may be developed to satisfy customer needs, but are not defined by customers. Therefore, answer a is incorrect. Objectives must be defined before the plans to meet those objectives can be determined. Answer b is also incorrect. Since the short-term plans support the long range objectives, they cannot be opposite to them. Answer d is false. Both short and long-term quality plans should support strategic objectives.

A comparison between the Cp and Cpk for a process would find which of the following to be true? a. The Cpk value is often larger than Cp b. The denominator of the Cp calculation is half that of the Cpk c. The Cp value does not account for centering d. Neither calculation requires a stable process

c. The Cp value does not account for centering

Three trainees were given the same lot of 50 pieces and asked to classify them as defective or non-defective with the following results...Using a level of significance of 0.050 to determine whether or not there is a difference in the ability of the three trainees to properly classify the parts, which of the following statements is true? a. The chi-square calculated value is 5.99 b. The critical value of the chi-square is 6.88 c. The null hypothesis is rejected d. No determination can be made, more date is required

c. The null hypothesis is rejected

In order to test whether the average output of a new machine is difference than the existing machine, a sample of ten pieces was taken from each. The calculated t value turned out to be 1.767. Using a level of significance of 0.10, and given that the variances are unknown, but considered equal, the conclusion is: a. The obtained t ratio does not fall within the critical region b. There was no significant difference between the means c. The null hypothesis was rejected d. The null hypothesis failed to be rejected

c. The null hypothesis was rejected

If the 95% confidence limits for mean m turn out to be 6.5 and 8.5: a. The probability is 0.95 that X-bar falls between 6.5 and 8.5 b. The probability is 0.95 that X falls between 6.5 and 8.5 c. The probability is 0.95 that the interval (6.5, 8.5) contains population mean d. 4 sigma = 8.5 - 6.5

c. The probability is 0.95 that the interval (6.5, 8.5) contains population mean

When comparing short term machine capability indexes to long term process capability indexes, one would expect to find: a. A plus and minus shift of 1.5 standard deviations b. The machine capability to be a lower number c. The process capability to be a lower number d. The machine and process capabilities to be virtually identical

c. The process capability to be a lower number

An X-bar chart has shown control for a long time. However, the points for the last fifty samples are all very near the center line on the chart. In fact, they are all within one sigma of the center line. This probably indicates that: a. A desirable situation has developed b. An undesirable situation may occur c. The process standard deviation has decreased during the last fifty samples d. The control limits are incorrectly computes

c. The process standard deviation has decreased during the last fifty samples

An engineer has conducted a year-end analysis (365 data points) of incoming materials by checking for dimensional variation. For one of the items (a ruler), the print dimension of length is to be 12.50 inches +/- 0.02 inches. The computer software indicated that the grand average was 12.52 inches and was statistically different from 12.50 inches. The engineer should do the following: a. Since the result is statistically significant, request the supplier to modify his equipment by 0.02 inches b. Follow the decision of the computer analysis, start rejecting the rulers c. The result is not of practical significance; the difference is too small to justify a change d. The supplier is to be notified via a corrective action request to correct his process

c. The result is not of practical significance; the difference is too small to justify a change

When performing an analytical study, which of the following statistical values would seldom be known? a. The true critical value b. The sample statistic c. The true population parameter d. The degree of uncertainty

c. The true population parameter

The use of schedule boards, tool boards, jidhoka devices, and kanban cards are describing what the Japanese consider to be a part of: a. 5S activities b. Delivery information c. The visual factory d. Standardized work instructions

c. The visual factory

A critical to quality (CTQ) tree can be of value to the Six Sigma project team. It can translate customer requirements to quantified requirements. This will allow the organization to focus on a problem. An example of a quantified requirement for a mail order publishing house is: a. Cycle time to print a book b. Inexpensive pricing c. The weight of a book not to exceed three pounds d. Exceptional copy quality

c. The weight of a book not to exceed three pounds

An organization is considering a kaizen blitz event for improving the system software functionality by replacing the network server and implementing a new software program. Which of the following is the best response to this consideration? a. This is an ideal application because the project would only take 3 to 5 days b. A minimum of 25 people should be involved because of the short duration of the change c. This is not a suitable application because of the complexity of and duration d. The change-over should be done on the weekend to avoid downtime

c. This is not a suitable application because of the complexity of and duration

The major difference between a kaizen and kaizen blitz activity is: a. The number of people involved b. Money c. Time d. The availability of data

c. Time

The use of the program evaluation and review technique (PERT) requires: a. The critical path to be known in advance b. Slack times to be added to the critical path c. Time estimates for each activity in the network d. Less data than a Gantt chart

c. Time estimates for each activity in the network

What is the definition of the mitigate step in the risk management process? a. To reduce the impact of any unforeseen event b. To hide any relevant risk studies from the stakeholders c. To make adjustments based on the deviations from planned actions d. To delegate disaster relief to the proper authorities

c. To make adjustments based on the deviations from planned actions

It is suspected that a process requiring a capability determination is not normal, but appears to be stable. The last action to take at this point, would be to: a. Advise the customer and request specification changes b. Reduce variation to the point that it does not matter c. Transform the data to that of a normal distribution d. Test the normality assumption using a chi-square test

c. Transform the data to that of a normal distribution

If a process is out of control, the theoretical probability that a single point on the X-bar chart will fall between one sigma and the upper control limit is: a. 0.2240 b. 0.1587 c. Unknown d. 0.3414

c. Unknown

A company struggling with low performance in terms of quality, profitability, and productivity is considering Six Sigma initiative. A decision to proceed would be considered: a. Smart, they have a lot of low lying fruit b. Unwise, they probably can't afford the effort c. Unwise, they need to attend to basic activities first d. Smart, they obviously need the Six Sigma structure

c. Unwise, they need to attend to basic activities Most authorities agree that the company in question should consider more fundamental building blocks first.

A study on best customer satisfaction practices suggests which principle? a. Use proper sampling methods to collect data b. Use consultant designed survey forms c. Use multiple instruments to collect data d. Use good equipment for sampling

c. Use multiple instruments to collect data

When plotting a multi-vari chart on graph paper, what metric is used for the vertical scale? a. Time b. Count data c. Variable data d. Percentages

c. Variable date

In performing an analysis of variance for a single factor experiment, a fundamental assumption which is made is that the factor: a. Means are equal b. Means are unequal c. Variances are equal d. Variances are unequal

c. Variances are equal

Identify the confidence interval calculation that is most likely to be non-symmetrical. a. Means for larger samples b. Means for small samples c. Variation confidence interval d. Proportion confidence interval

c. Variation confidence interval

Which of the following forms of muda affects people more than product quality? a. Overproduction b. Excess inventory c. Waiting d. Transport

c. Waiting The muda of waiting is the best choice. The other answers have more direct impact on the product.

When giving instructions to those who will perform a task, the communication process is completed: a. When the worker goes to his work station to do the task b. When the person giving the instruction has finished talking c. When the worker acknowledges these instructions by describing how he/she will perform the task d. When the worker says that he/she understands the instructions

c. When the worker acknowledges these instructions by describing how he/she will perform the task

What charting process uses data for multiple plot points? a. np charts b. X - bar - R cahrts c. mX bar - mR charts d. Run charts

c. mX bar - mR charts

Which of the following statistical distributions can be used to compare sample means? a. Chi-square distribution b. Normal distribution c. t distribution d. Exponential distribution

c. t distribution

The average number of flaws in large plate grass is 0.25 per pane. The standard deviation of this Poisson distribution is: a. 0.25 b. 0.05 c. 0.75 d. 0.50

d. 0.50

Given the following information: Probability of 1 or more defects = 0.69 Probability of 2 or more defects = 0.34 Probability of 3 or more defects = 0.12 Probability of 4 or more defects = 0.03 What is the probability of 2 or fewer defects? a. 0.34 b. 0.69 c. 0.46 d. 0.88

d. 0.88

The probability of a train arriving on time and leaving on time is 0.8. The probability of the same train arriving on time is 0.84. The probability of this train leaving on time is 0.86. Given the train arrived on time, what is the probability it will leave on time? a. 0.93 b. 0.84 c. 0.88 d. 0.95

d. 0.95

The result of a designed experiment are to be analyzed using a chi-square test. There are five treatments under consideration and each treatment falls under two categories (success or failure). The calculated value of chi-square is compared to the tabulated chi-square with how many degrees of freedom? a. 10 b. 9 c. 5 d. 4

d. 4

The SIPOC process maps stands for suppliers, inputs, process, outputs, and customers. It provides a view of the process that contains approximately how many steps? a. 21-40 steps b. 16-20 steps c. 8-15 steps d. 4-7 steps

d. 4-7 steps

What is the mean height of five men who have the following heights? 5'6, 5'9, 5'4, 5'11, 5'8. a. 6'0 b. 5'5 c. 5'6 1/2 d. 5'7 3/5

d. 5'7 3/5

In a full factorial experiment with 4 factors at 3 levels each, how many trials are required? a. 24 b. 12 c. 64 d. 81

d. 81

The grades of a student on six examinations were 84, 91, 72, 68, 87, and 78. Find the median of the grades: a. 84 b. 78 c. 80 d. 81

d. 81

If 87 observations from a process were to be plotted on a histogram, the rule of thumb would suggest which number of intervals across the range of data? a. 19 b. 4 c. 12 d. 9

d. 9

A small positive change truly exists between a test trial and the current process. However, the sample from the test trial happens to demonstrate a radical improvement. What type of decision would be probably be made? a. A type II error b. A 1-alpha decision c. A type I error d. A 1-beta decision

d. A 1-beta decision

Studies have shown that the most effective method to communicate information is: a. Oral b. Written c. Non-verbal d. A combination of methods

d. A combination of methods

What is jidhoka? a. A totally automated process to reduce defectives b. A device that identifies good parts c. A technique which separates defectives from good parts d. A device that stops the machine whenever a defective is produced

d. A device that stops the machine whenever a defective is produced

A commonly reported problem associated with Six Sigma projects deals with: a. A failure to complete any project charter documentation b. A desire to complete projects on time c. A requirement that projects must be at least $100,000 in value d. A lack of business impact for the company

d. A lack of business impact for the company

Non-locational data might be found on which data sheets? a. A defect diagram check sheet b. A measles chart c. A concentration chart d. A recording check sheet

d. A recording check sheet

A technique used to discover the most probable important factors in an experimental trial is called: a. Full test coverage b. A second order experiment c. A simplex design d. A screening experiment

d. A screening experiment

The term "metrics" most frequently refers to: a. A unit of measurement b. The metric system c. The science of weights and measurements d. An evaluation method

d. An evaluation method This is a definition question. Answers a and b refer to a traditional form of measurement such as the metric system and are incorrect. Answer c is the definition for metrology. The modern definition of "metrics" is a form of measurement or evaluation (answer d).

Which of the following problems would most likely appear in the X-bar control chart pattern shown? a. A varying environmental temperature b. A tired operator c. An incorrect calculation of the control limits d. An output process alternatively fed by different input processes

d. An output process alternatively fed by different input processes

Having extensive industry knowledge makes upper management "experts" in customer needs and desires. To win in the marketplace, they should: a. Authorize many new products as soon as possible b. Start with a new marketing plan fitting customer needs c. Develop a strategic plan for new products d. Ask for help, because they often don't really know the customer

d. Ask for help, because they often don't know the customer

In the construction of a Pareto chart for the number of defects in a book binding operation, four categories of defects were recorded, as identified in the following table. The correct listing for the Pareto chart categories from left to right would be: a. Emulsion, other, sewing, and grease b. Grease, sewing, other, and emulsion c. Grease, sewing, emulsion, and other d. Emulsion, sewing, grease, and other

d. Emulsion, sewing, grease, and other

The key difference between internal and external customers is: a. Their interest in the product or service b. Internal customers can influence the design of the product c. External customers usually influence the design of the product d. External customers best determine the true quality of the product

d. External customers best determine the true quality of the product

When should an X - MR chart be used? a. When the number of defectives data is being monitored b. When an exceptionally large run size is expected c. When range data is unreliable d. For destructive testing applications

d. For destructive testing applications

Effective control chart subgrouping includes which of the following actions? a. Group product so that subgroups are as heterogeneous as possible b. Group product produced over a long period of time c. Group product to achieve a minimum opportunity for variation between subgroups d. Group product to be representative of production over a short time period

d. Group product to be representative of production over a short time period

A process has been performing satisfactory for some time. An improvement is required. Your response is to: a. Direct the workforce to be more careful in their work b. Issue a slogan "do it right the first time" c. Identify the special causes to correct the process d. Identify the common causes conditions to correct the process

d. Identify the common causes conditions to correct the process

What is the relation between resolution III experiments and confounded response? a. In resolution III experiments, there are no confounded interactions b. In resolution III experiments, only interactions are confounded c. In resolution III experiments, all factors are confounded d. In resolution III experiments, main effects and two factor interactions are confounded

d. In resolution III experiments, main effects and two factor interactions are confounded

There are many advantages to the adoption of mistake proofing in the workplace. What could possibly be a downside to this approach? a. Considerable training is required for the workforce b. Many employees see it as a form of "nanny state" c. In most cases, production operations actually take longer d. In some situations, technical or engineering assistance is required

d. In some situations, technical or engineering assistance is required

An X-bar control chart is based on a sample size of 4. An operator mistakenly plots the value of a single observation on the control chart. This point: a. Will not cause any mis-judgements if the process is in control b. Will always be plotted near the center line of the control chart c. Will cause the associated R chart to show an out of control condition d. Increases the probability of the process being labeled out of control

d. Increases the probability of the process being labeled out of control

When finding a confidence interval for mean mu, based on a sample size of n: a. Increasing n increases the interval b. Having to use Sx instead of n decreases the interval c. The larger the interval, the better the estimate of mu d. Increasing n decreases the interval

d. Increasing n decreases the interval

While plotting a control chart, it is noted that two of the last three points are than 2 sigma, four out of the last points are greater than 1 sigma, and eight successive points are on one side of the center line. This information suggests that one should: a. stop the process immediately b. take more readings and continue to plot c. write a discrepancy notice to the supervisor d. investigate to determine what has changed

d. Investigate to determine what has changed

The advantages of an effective quality cost measurement system include which of the following? a. It provides an effective vehicle for Kano analysis b. It provides the solution to many quality problems c. It eliminates the need for separate management corrective actions efforts d. It aligns many company and quality costs

d. It aligns many company and quality costs

What input categories are commonly used in cause-and-effect diagrams? a. Maintenance, method, mission b. Manpower, motivation, marking c. Machine, mission, metrology d. Material, machine, measurement

d. Material, machine, measurement

In almost every definition of standard work, one would find some combination of: a. Current state, future state, and planning b. Signal, process, and schedule c. Source, buffer, and production d. Materials, workers, and machinery

d. Materials, workers, and machinery

Which of the following are risk handling activities? a. Interviews, lessons learned, plan review b. Needs, resources, requirements c. Simulations, decision analysis d. Mitigation, avoidance, acceptance

d. Mitigation, avoidance, acceptance

An X-bar - R chart has been in control for sometime. If the range suddenly and significantly increases, the mean will: a. Usually increase b. Stay the same c. Always decrease d. Occasionally show out of control of either limit

d. Occasionally show out of control of either limit

A Six Sigma improvement team may be required to analyze customer data in order to define a project or the results of an improvement. Which of the following tools would be of greatest value? a. Conflict resolution b. Brainstorming and multivoting c. Nominal group techniques and voting d. Pareto and statistical analysis

d. Pareto and statistical analysis

A three-prong 110 volt electrical plug is an example of: a. Yin and yang because of alternating current wires b. The jidoka self-inspection process c. Heijunka design improvement and optimization d. Poka-yoke permitting only proper plug insertion

d. Poka-yoke permitting only proper plug insertion

Identify the major variation classifications specifically addressed in multi-vari studies a. Cylinder, batch, flat b. Piece-to-piece, batch-to-batch, within piece c. Temporal, top to bottom, side to side d. Positional, cyclical, temporal

d. Positional, cyclical, temporal

In the planning of a new major manufacturing program, the greatest quality effort should be put logically in: a. Inspection of product b. Nondestructive testing equipment c. Nonconformance to specifications d. Prevention of occurrence of substandard

d. Prevention of occurrence of substandard

What is the effect of management tampering with process capability? a. Process capability will improve if management uses a motivational approach b. Process capability will improve if management punishes the poor quality offenders c. Process capability will deteriorate if management does not act promptly d. Process capability will deteriorate if management mandates frequent adjustments

d. Process capability will deteriorate if management mandates frequent adjustments

In the automotive sector, the APQP identifies what control plan for the early development stages of a product? a. Production b. Pre-launch c. Subprocess d. Prototype

d. Prototype

Which of the following techniques has proven useful in translating customer needs into product design features? a. Changing perceptions b. Customer service principles c. Confrontation and problem solving d. Quality function deployment

d. Quality function deployment Answers a and c are customer conflict resolution techniques. Answer b addresses service principles, not the question at hand. The role of quality function deployment (QFD) is to translate customer needs into design features.

Which of the following is a non-value added activity? a. Design reviews b. Vendor assessments c. Inventory reductions d. Receiving inspection to ensure incoming quality

d. Receiving inspection to ensure incoming quality Receiving inspection is non-value added since the vendor should be shipping quality products. The other items are, or can be value added.

The house of quality is used to translate customer wants into engineering design variables. The linking or prioritizing of customer wants into engineered values occurs in what element? a. Competitive analysis b. Conflict analysis c. Technical review d. Relationship matrix

d. Relationship matrix

The theory of constraints concentrates mainly on: a. Understanding customer needs b. Developing a value stream map c. Achieving on-time goals d. Removing process bottleneck

d. Removing process bottlenecks The theory of constraints concentrates on removing constraints or bottlenecks in a process to increase throughput. Achieving on-time goals may not increase throughput.

Defining the sample space S as {rock, book, cigar, guitar, dog} what is the compliment of {cigar, dog}? a. Rock, book, cigar, guitar, dog b. Cigar, guitar, dog c. Dog d. Rock, book, guitar

d. Rock, book, guitar

Two micrometers are used to measure the same quality characteristic (thickness in this case). The micrometer at headquarters has more decimal places than the one being used at the plant. The micrometer at headquarters is more: a. Accurate b. Precise c. Advanced d. Sensitive

d. Sensitive

In preparation of a project, efforts should be made to identify and involve various parties affected by the planned changes. These other parties are known as: a. Process owners b. Champions c. Team leaders d. Stakeholders

d. Stakeholders

Pallets of product are staged in a warehouse prior to shipment. There are indications that container damage is occurring disproportionately at aisle and row end locations. The best method of data collection to confirm this theory would be? a. Random sampling b. Sequential sampling c. Skip-lot sampling d. Stratified sampling

d. Stratified sampling

What design structure uses the following four main phrases in the design process? Clarify the task, develop a conceptual design, refine the design, generate the final detailed design: a. Set-based design b. Pugh analysis c. Critical parameter design d. Systematic design

d. Systematic design The question is describing the step-by-step German methodology known as systematic design.

If you choose to look at any business enterprise on a main level basis, which of the following categories would be integrated into other KPIV and KPOV areas? a. Process b. Operations c. Business d. Technological

d. Technological Although the variables themselves may differ, process, operations, and business levels have key input and output variables and identified as such. A technological level is not separately identified. It is integrated into the major three categories.

In Plackett - Burman geometric designs, one would expect: a. That each interaction effect is confounded with two main effects b. That each interaction effect would not be confounded c. That only pure main effects will be determined d. That each interaction effect will be confounded with exactly one main effect

d. That each interaction effect will be confounded with exactly one main effect

A test of hypothesis was performed, although the desired level of confidence was not established prior to performing the calculations. A p-value of 0.0416 was found. One would most likely conclude: a. That a statistically significant difference has not been proven b. To fail to reject the null hypothesis c. The null hypothesis can neither be rejected or fail to be rejected d. That there is a statistically significant difference

d. That there is a statistically significant difference

The upper and lower specification limits for a certain product are 7.3 pounds and 7.0 pounds. Actual data indicates that the product is current at an average of 7.465 pounds with a standard deviation of 0.03039 pounds. The calculations indicate Cp = 1.645 and Cpk = -1.81. What conclusions can be made about the process? a. There is something wrong with the calculations; a negative Cpk is not possible b. The specifications must be unrealistically set c. The process is close to 6 sigma; the negative Cpk is irrelevant d. The Cp and Cpk values indicate that the process is not centered

d. The Cp and Cpk values indicate that the process is not centered

What is a major distinction between the CPM and PERT methods in the evaluation of project performance? a. Only the PERT method can be displayed on a Gantt chart b. The PERT technique allows for easier crashing of project time c. The PERT technique permits network relationships but CPM does not d. The PERT technique is event oriented, while CPM is activity oriented

d. The PERT technique is event oriented, while CPM is activity oriented

One would say that the kanban method would be most closely associated with: a. The elimination of non-value added activities in the process b. The development of a future state process stream map c. Making problems visible in a process, thus clarifying targets for improvement d. The control of material flow

d. The control of material flow

A Six Sigma Project has progressed to the point that a control plan is required. Control plan activities can be considered closed after which of the following? a. A process owner is named for the control plan b. A responsible engineer is designated c. The cross functional team signs off on the control plan d. The control plan is a living document and is rarely closed

d. The control plan is a living document and is rarely closed

Process capability analysis is often defined as: a. The ability to make the process reliable and maintainable b. The inherent variability of items produced by the process c. The variability allowed by the specification limits d. The determination that the process can meet the product specifications as intended

d. The determination that the process can meet the product specifications as intended

A disadvantage of computer software driven project management methods includes: a. The ability to analyze "what-if" options b. An automatic calculation of the critical path c. A determination of the effects of the actual results on the project d. The higher training costs

d. The higher training costs

If a control chart has UCL = 45.1 and USL = 45.6 and the last sample has values of 44.6, 45.7, and 44.8, one can conclude: a. The process is within specification and control b. The process is within specification and out of control c. The process is out of specification but in control d. The process is out of specification and out of control

d. The process is out of specification and out of control

During a project work breakdown structure, a number of planning activities occur. Which of the following items is included? a. The project objective is defined b. The work is divided into smaller activities c. The project is divided into a series of sequential steps d. Determining the RTY

d. The work is divided into smaller activites

According to Pande, which of the following is a legitimate reason for embracing a Six Sigma effort? a. The company currently has an effective improvement effort b. Current changes are overwhelming the company resources c. Potential gains aren't sufficient to fund six sigma d. There are difficulties in meeting customer requirements

d. There are difficulties in meeting customer requirements Pande suggests that answers a, b, and c are valid reasons to not undertake Six Sigma improvement. Answer d is a valid reason to undertake Six Sigma.

What is the practical use of the F distribution? a. To study the equality of two means b. To study the equality of goodness of fit data c. To study the equality of one mean and one variance d. To study the equality of two variances

d. To study the equality of two variances

A process map is used to accomplish which of the following? a. Display a dynamic picture of process performance behavior b. Focus attention on process problems in priority order c. Diagram possible problems causes in a process d. Track products, operator actions, or administrative procedures

d. Track products, operator actions, or administrative procedures

What key step would follow an evaluation of current practices against a benchmark? a. Identifying key performance factors b. Selecting performance criteria based on priorities c. Determining a leader in a critical performance area d. Undertaking significant changes in performance

d. Undertaking significant changes in performance

The organization's customer service program can be enhanced in many ways. One of the ways would be: a. Provide better procedures for customer service personnel b. Restrict access to customer data c. Have supervisors available to answer more questions d. Utilize employee involvement

d. Utilize employee involvement

The Poisson distribution can be used to approximate the binomial distribution under which of the following conditions? a. When p is equal to or larger than 0.1 and the sample size is large b. When p is equal to or larger than 0.1 and the sample size is small c. When p is equal to or smaller than 0.1 and the sample size is small d. When p is equal to or smaller than 0.1 and the sample size is large

d. When p is equal to or smaller than 0.1 and the sample size is large

The matrix diagram is used to show the relationship between two variables. Matrices can be developed in several ways. Which of the following matrix types illustrates relationships in three planes? a. L-type b. T-type c. C-type d. Y-type

d. Y-type

Which of the following is a valid null hypothesis? a. p > 1/8 b. mu < 98 c. The mean of population A is not equal to the mean of population B d. mu = 110

d. mu = 110

What control chart is recommended to plot defectives using a fixed sample size? a. u chart b. c chart c. p chart d. np chart

d. np chart

A Six Sigma team is investigating the sources of variation in fabric rolls. All rolls are different in length. The variable of interest is the number of holes per unit of length. Which control chart will work best for this situation? a. X-bar and R chart b. p chart c. c chart d. u chart

d. u chart


Ensembles d'études connexes

Chapter 1: Sole Proprietorship and Franchises

View Set

CHAPTER 15-29: GERIATRIC NURSING 1ST SEM FINALS

View Set

Vet 265: Vaccines in companion animals

View Set